The water pressure on a submerged object is given by P = 64d, where P is the pressure in pounds per square foot, and d is the depth of the object in feet.
a. solve the formula for d.
b. Find the depth of a submerged object if the pressure is 672 pounds per square foot.
a. Solving for 'd' as the subject of formula gives d = P/64
b. The depth of a submerged object if the pressure is 672 pounds per square foot is 10. 5 feet
What is a function?A function can be defined as an expression, law or rule that shows the relationship between two variables.
These variables are namely;
Independent variableDependent variableGiven the function as;
P = 64d
Where;
P is the pressure in pounds per square foot d is the depth of the object in feetNow, let's make the variable 'd' the subject of formula
Divide both sides of the equation with the coefficient of the variable 'd' which is 64. This is done so that the variable stands alone.
We have;
P/64 = 64d/64
d = P/ 64
If the pressure, P equals 672 pounds per square foot
Substitute the value of the variable into the formula, we have;
d = P/64
Then,
d = 672/64
Find the quotient
d = 10. 5 feet.
Thus, the value is 10. 5 feet
Learn more about functions here:
https://brainly.com/question/2328150
#SPJ1
A 2-column table with 5 rows. Column 1 is labeled Number of Apples with entries 0, 1, 2, 3, 4. Column 2 is labeled Number of Slices with entries 0, 6, 12, 18, 24.
Use the table to find the constant of proportionality and the equation that represents the relationship.
Let x = number of apples and y = number of slices.
The constant of proportionality is
.
The equation for this relationship is
.
plss help
Answer:
the answer is c,Yes, because the ratios for each input and output are equivalent.
Step-by-step explanation:
The first one is 6
The second one is is y= 6x
A rectangular swimming pool is 4 ft deep. One side of the pool is 4.5 times longer than the other. The amount of water needed to fill the swimming pool is 1800 cubic feet.
Find the dimensions of the pool.
Answer:
19 x 23.5 x 4
Step-by-step explanation:
4 * x * (x + 4.5) =1800
Divide both sides by 4
x * (x + 4.5) = 450
Use distributive property
x^2 + 4.5x = 450
Move all terms to one side
x^2 + 4.5x - 450 = 0
Use quadratic formula to solve a = 1, b = 4.5, c = -450
-4.5 +- \(\sqrt{4.5^2 - 4 * 1 * -450}\)/2
=-4.5 + - \(\sqrt{1820.25}\) / 2
= -4.5 +- 42.664 / 2
=19.08 and -23.582. You can't have a negative length, so it is 19.08, or 19
19 x 23.5 x 4
ASAP PLS HELP thanks!
Answer:
the first step is combining like terms
the second step is the subtraction property of equality
and the last is the answer so I guess given because it's not the others
Does anyone know this? Needed ASAP
Answer:
50 square units
Step-by-step explanation:
Using the formula provided, \(a = \frac{1}{2}h(b_1+b_2)\), substituting the height (5), b1 (8) and b2 (12), we can solve for the formula.
\(a = \frac{1}{2}\cdot5(8+12)\\\\a = \frac{1}{2}\cdot5(20)\\\\a = \frac{1}{2}100\\\\a = 50\)
Hope this helped!
Answer:
50 units²
Step-by-step explanation:
The area of a trapezoid can be found using the following formula.
\(A=\frac{1}{2}h(b_{1} +b_{2} )\)
The height of the trapezoid is 5 units. The bases are 8 units and 12 units.
\(h= 5 \\b_{1} = 8\\b_{2} = 12\)
Substitute the values into the formula.
\(A=\frac{1}{2}*5(8+12)\)
Solve inside the parentheses. Add 8 and 12.
8+12=20
\(A=\frac{1}{2}*5(20)\)
Multiply 5 and 20.
5*20= 100
\(A=\frac{1}{2}*100\)
Multiply 1/2 and 100 or divide 100 by 2.
100 * 1/2= 50 or 100/2= 50
\(A= 50\)
Add appropriate units. For this problem, the units are units².
\(A= 50 units^2\)
The area of the trapezoid is 50 square units.
The science center has 300 people in attendance on Tuesday. This is 150 percent of the attendance on Monday. Natasha is trying to figure out how many people were in attendance on Monday. Her work is shown below.
StartFraction part Over whole EndFraction = StartFraction 150 Over 100 EndFraction
StartFraction 150 times 2 Over 100 times 3 EndFraction = StartFraction 300 Over question mark EndFraction
Since 150 times 2 = 300, then 100 times 3 or 300 people were in attendance on Monday.
What mistake did Natasha make?
Natasha should have multiplied 100 by 2.
Natasha should have divided 150 by 3.
Natasha should have added 150 and 300.
Natasha should have multiplied 300 and 150 percent.
Answer:
Natasha should have multiplied 100 by 2.
Step-by-step explanation:
part/whole = 150/100
(150 * 2)/(100 * 2) = 300/? is correct.
Answer: Natasha should have multiplied 100 by 2.
2) Ayanda wants to invest R200 000. The bank offers him 2 options for his
6 year investment.
Option 1: 12% Simple interest p.a.
Option 2: 9,5% Compound interest p.a.
4.2.1) Calculate the return on Ayanda's investment using Option 1.
●
●
4.2.2) Calculate the return on Ayanda's investment using Option 2.
4.2.3) Which option will render the most money?
Answer:
4.2.1) R140 000
4.2.2) R144 758.28
4.2.3) Option 2
Step-by-step explanation:
To calculate the return on Ayanda's investment using Option 1, we can use the simple interest formula.
\(\boxed{\begin{minipage}{7 cm}\underline{Simple Interest Formula}\\\\$ I =Prt$\\\\where:\\\\ \phantom{ww}$\bullet$ $I =$ interest accrued \\ \phantom{ww}$\bullet$ $P =$ principal amount \\ \phantom{ww}$\bullet$ $r =$ interest rate (in decimal form) \\ \phantom{ww}$\bullet$ $t =$ time (in years) \\ \end{minipage}}\)
Given values:
P = R200 000r = 12% = 0.12t = 6 yearsSubstitute the given values into the formula and solve for I:
\(I=200000 \cdot 0.12 \cdot 6\)
\(I=24000 \cdot 6\)
\(I=144000\)
Therefore, the return on Ayanda's investment using Option 1 is R144000.
\(\hrulefill\)
To calculate the return on Ayanda's investment using Option 2, we can use the compound interest formula.
\(\boxed{\begin{minipage}{7 cm}\underline{Annual Compound Interest Formula}\\\\$ I=P\left(1+r\right)^{t}-P$\\\\where:\\\\ \phantom{ww}$\bullet$ $I =$ interest accrued \\ \phantom{ww}$\bullet$ $P =$ principal amount \\ \phantom{ww}$\bullet$ $r =$ interest rate (in decimal form) \\ \phantom{ww}$\bullet$ $t =$ time (in years) \\ \end{minipage}}\)
Given values:
P = R200 000r = 9.5% = 0.095t = 6 yearsSubstitute the given values into the formula and solve for I:
\(I=200000(1+0.095)^6-200000\)
\(I=200000(1.095)^6-200000\)
\(I=200000(1.72379142...)-200000\)
\(I=344758.28426...-200000\)
\(I=144758.28426...\)
\(I=144758.28\)
Therefore, the return on Ayanda's investment using Option 2 is R144758.28.
\(\hrulefill\)
Comparing the returns from both options, we find that Option 1 offers a return of R144000, while Option 2 offers a return of R144758.28. As R144758.28 > R144000, then Option 2 will render the most money for Ayanda's investment.
Freemont run club surveyed a random sample of 30 of their members about their running habits
The reasonable estimate would be 25
How to solve for the estimateWe have to solve this using the rule of 3
9 out of 30 members said that they run more than 5 days a week.
We will form the equations
when 9 ran = 30 menbers
when n ran = 84 members
we will then cross multiply
84 x 9 = 30 x n
n = 25.2
When we round this, the reasonable estimate would be 25
Read miore on survey here:https://brainly.com/question/25257437
#SPJ1
Complete question
Freemont Run Club surveyed a random sample of 30 of their members about their running habits. Of the members surveyed, 9 said that they run more than 5 days a week.
There are 84 Freemont Run Club members.
Based on the data, what is the most reasonable estimate for the number of Freemont Run Club members who run more than 5 days a week?
An equation is shown below:
5(2x − 3) = 5
Part A: How many solutions does this equation have? (4 points)
Part B: What are the solutions to this equation? Show your work. (6 points)
Your answer:
Answer:
this equation has 1 answer
Step-by-step explanation:
x = 2
Caleb started with 50 bacteria for his science experiment and they doubled in one day. Jenna also started with 50 bacteria for her experiment and after one day, her number of bacteria was equal to 50 to the second power. Choose the statement that is true for the number of bacteria after one day.
A.
Caleb's experiment had 100 bacteria and Jenna's experiment had 250 bacteria.
B.
Caleb's experiment had 100 bacteria and Jenna's experiment had 2,500 bacteria.
C.
Caleb's experiment had 500 bacteria and Jenna's experiment had 500 bacteria.
D.
Answer:
I'm pretty sure the answer is A or what ever letter that says caleb experiment had 100 and jenna had 250
6.
A mug is 3/7
full. The mug contains 1/2
of a cup of water. Find
the capacity of the mug. Write the
answer as a fraction or mixed
number in simplest form.
contains
The capacity of the mug is 7/6 cups.
What is the capacity of the mug?
Given,
A mug is 3/7 full.
The mug contains 1/2 of a cup of water.
Solution:
Let x be the water.
Water the mug has = 3/7 of x
= 3/7x
Since the water in the mug is 1/2 cup,
3/7x = 1/2
x = 1/2 ÷ 3/7
= 1/2 × 7/3
= 7/6
Therefore,
The capacity of the mug is 7/6 cups.
To learn more about capacity, refer
https://brainly.com/question/13484626
#SPJ9
29. Assertion :7√5, √2+21 are the irrational number. Reason: every integer is an rational number 30.
The assertion is true. Both 7√5 and √2 + 21 are irrational numbers because they cannot be expressed as fractions.
The assertion is true. Both 7√5 and √2 + 21 are irrational numbers.
An irrational number is defined as a number that cannot be expressed as a fraction of two integers and has an infinite non-repeating decimal representation.
In the case of 7√5, the square root of 5 is an irrational number because it cannot be expressed as a fraction. Multiplying it by 7 does not change its irrational nature.
Similarly, √2 is also an irrational number because the square root of 2 cannot be expressed as a fraction. Adding 21 to √2 does not alter its irrationality.
The reason provided, that every integer is a rational number, is not relevant to the given assertion. While it is true that every integer is a rational number because it can be expressed as a fraction (e.g., 3 can be written as 3/1), it does not contradict the fact that 7√5 and √2 + 21 are irrational numbers.
In conclusion, the assertion is valid, and both 7√5 and √2 + 21 are irrational numbers.
For more questions on fractions
https://brainly.com/question/78672
#SPJ8
825 use each digit once. make the smallest 3digit number
Step-by-step explanation:
Given: To make smallest 3-digit number of 825.
To find: The smallest 3-digit number of 825.
Solution: We can make the smallest 3-digit number of 825 by separating the numbers and arranging it to ascending order. The given number is 825. ...
Final answer: The smallest 3-digit number of 825 is 258.
hope it helps
Answer:
258
Step-by-step explanation:
We are given 3 numbers:
8 2 5
And we are asked to find the smallest 3 digit number using those 3 digits above.
To make the smallest number, place the numbers in value from least to greatest:
2 5 8
This is your 3 digit number: 258.
Hope this helps! :)
Help me ASAP plsssssssssssssssssssssssssssss
Answer:
B
Step-by-step explanation:
Answer: B. Recycling
Step-by-step explanation: Recycling is turning something like newspapers and making them into new products.
Mark Brainliest if you want!
find the value of x and the mesasurement of angle axc
Answer:
x = 4 , ∠ AXC = 150°
Step-by-step explanation:
∠ 1 and ∠ 2 form the angle AXC , that is
∠ AXC = ∠ 1 + ∠ 2 , then
6(6x + 1) = 102 + 10x + 8
36x + 6 = 10x + 110 ( subtract 10x from both sides )
26x + 6 = 110 ( subtract 6 from both sides )
26x = 104 ( divide both sides by 26 )
x = 4
Then by substituting x = 4
∠ AXC = 6(6x + 1) = 36x + 6 = 36(4) + 6 = 144 + 6 = 150°
) How many arrangements of the 26 different letters of the alphabet (with no repeats) are there in which all the consonants appear before all the vowels (a,e,i,o,u)
Answer:the answer is 21 with np repeats
Step-by-step explanation:
Keller Construction is considering two new investments. Project E calls for the purchase of earthmoving equipment. Project H represents an investment in a hydraulic lift. Keller wishes to use a net present value profile in comparing the projects. The investment and cash flow patterns are as follows: Use Appendix B for an approximate answer but calculate your final answer using the formula and financial calculator methods.
Based on the net present value profile, Project H has a higher NPV than Project E.
To compare the net present value (NPV) of Project E and Project H, we need to calculate the present value of cash flows for each project and determine which one has a higher NPV. The cash flow patterns for the two projects are as follows:
Project E:
Initial investment: -$100,000
Cash flows for Year 1: $40,000
Cash flows for Year 2: $50,000
Cash flows for Year 3: $60,000
Project H:
Initial investment: -$120,000
Cash flows for Year 1: $60,000
Cash flows for Year 2: $50,000
Cash flows for Year 3: $40,000
To calculate the present value of cash flows, we need to discount them using an appropriate discount rate. The discount rate represents the required rate of return or the cost of capital for the company. Let's assume a discount rate of 10%.
Using the formula method, we can calculate the present value (PV) of each cash flow and sum them up to obtain the NPV for each project:
For Project E:
PV = $40,000/(1 + 0.10)^1 + $50,000/(1 + 0.10)^2 + $60,000/(1 + 0.10)^3
PV = $36,363.64 + $41,322.31 + $45,454.55
PV = $123,140.50
For Project H:
PV = $60,000/(1 + 0.10)^1 + $50,000/(1 + 0.10)^2 + $40,000/(1 + 0.10)^3
PV = $54,545.45 + $41,322.31 + $30,251.14
PV = $126,118.90
Using the financial calculator method, we can input the cash flows and the discount rate to calculate the NPV directly. By entering the cash flows for each project and the discount rate of 10%, we find that the NPV for Project E is approximately $123,140.50 and the NPV for Project H is approximately $126,118.90.
For more such questions on present value profile visit:
https://brainly.com/question/30390056
#SPJ8
From the equation, find the axis of symmetry of the parabola.
y=-4x² + 24x-35
a. X=1
b. x=-1
PD
Ο Α
OB
O C
OD
C.
d.
x=3
X=-3
Please select the best answer from the choices provided
The equation for the axis of symmetry of the parabola y = - 4 x² + 24 x - 35 is given by x = 3.
We know that the method to find the axis of the symmetry of the parabola is given by:
x = - b / 2 a
We have the general equation of the parabola as:
y = a x² + b x + c
We have the equation of the parabola as:
y = - 4 x² + 24 x - 35
Comparing from this equation, we get that:
a = - 4
b = 24
c = - 35
Now, substituting the values, we get that:
x = - 24 / 2 (- 4)
x = - 24 / - 8
x = 24 / 8
x = 3
Therefore, the equation for the axis of symmetry of the parabola y = - 4 x² + 24 x - 35 is given by x = 3.
Learn more about parabola here:
https://brainly.com/question/4061870
#SPJ9
evaluate the expression 4 ÷ 1/3
Answer:
12
Step-by-step explanation:
4 divided by 1 third is 12
Calculate the equivalent ratio 1.25 : 3.75 : 7.5
The equivalent ratio of 1.25 : 3.75 : 7.5 is 5 : 15 : 30.
To calculate the equivalent ratio of 1.25 : 3.75 : 7.5, we need to find a common multiplier that can be applied to all the numbers in the ratio to make them whole numbers. In this case, the common multiplier is 4 because it can be multiplied to each number to eliminate the decimals.
By multiplying each number in the ratio by 4, we get:
1.25 * 4 = 5
3.75 * 4 = 15
7.5 * 4 = 30
So the equivalent ratio of 1.25 : 3.75 : 7.5 is 5 : 15 : 30.
This means that the relative sizes or quantities represented by the original ratio are maintained in the equivalent ratio. For example, if we had 1.25 units of something, it would be equivalent to 5 units in the new ratio, and if we had 7.5 units, it would be equivalent to 30 units in the new ratio.
To more on ratio:
https://brainly.com/question/12024093
#SPJ8
dsfdewasedq24q2rwe3rq24rqweretretrrawsweqaeqw2e3q2wqewwasedq24q2rwe3rq24rqweretretrrawsweqaeqw2e3q2wqew
Answer:
???
Step-by-step explanation:
A computer training institute has 625 students that are paying a course fee of $400. Their research shows that for every $20 reduction in the fee, they will attract another 50 students. Which equation could be used to represent this situation, where x is the course fee and R(x) is the total revenue?
R(x) = −2.5x2 + 1625x
R(x) = −3x2 + 1650x
R(x) = 3x2 − 1650x
R(x) = 2.5x2 − 1625x
The equation that could be used to represent this situation, where x is the course fee and R(x) is the total revenue, is: R(x) = 250000 + 375x - 2.5x²
What is an Equations?Equations are mathematical statements with two algebraic expressionsοn either sideοf an equals (=) sign. It illustrates the equality between the expressions writtenοn the left and right sides. To determine the valueοf a variable representing an unknown quantity, equations can be solved. A statement is not an equation if there is no "equal to" symbol in it. It will be regarded as an expression.
N(x) = 625 + 2.5x
The revenue R(x) will be the productοf the numberοf students enrolled and the fee charged per student. The fee charged per student will be (400 - x) dollars. So, the revenue function can be represented as:
R(x) = (625 + 2.5x)(400 - x)
Simplifying the expression, we get:
R(x) = 250000 + 375x - 2.5x²
Therefore, the equation that could be used to represent this situation, where x is the course fee and R(x) is the total revenue, is:
R(x) = 250000 + 375x - 2.5x²
Learn more about equations, by the following link
brainly.com/question/2972832
#SPJ1
i dont understand this
We can Add 7 black beads to make ratio 3 : 1.
Since we can only change the number of black beads, decide how many black beads you will add based on how many white beads there are.
There are three white beads in the picture.
Total beads we will have (b meaning black)b : 3
Ratio black : white beads 3 : 1
Use the common ratio, which is a number that both sides of the original ratio multiply by to get to the new ratio.
Find common ratio by dividing total by ratio white beads: 3/1 = 3
Multiply ratio black beads by common ratio. 3 X 3 = 9
We need 9 black beads in total.
Check answer
9 : 3
Both sides divisible by 3; reduce ratio
= 3 : 1
Which is Correct ratio
Hence, There will be a total of 9 black beads, but we already have 2 black beads:
(9 total) - (2 original) = (7 to add)
Therefore , we need to add 7 black beads.
Learn more about the ratio visit:
https://brainly.com/question/12024093
#SPJ1
The excess of 15 over x is 10
The value of x that satisfies the given statement is 5.
What is the value of x?The given statement can be written as an equation:
The excess of 15 over x is 10
15 - x = 10
To solve for x, we can isolate the variable on one side of the equation. Adding x to both sides, we get:
15 - x + x = 10 + x
Simplifying the left side, the x terms cancel out:
15 = 10 + x
Subtracting 10 from both sides, we get:
15 - 10 = x
Simplifying the left side, we get:
5 = x
x = 5
Therefore, the value of x is 5.
Learn more about algebraic expressions here:
brainly.com/question/4344214
#SPJ1
what is the area of the triangle shown below?
Answer: Please provide the triangle
in circle P shown the measure of pqrs is 20 degrees what is the measure of p r s
From the properties of circle and chord ,
The angle RSQ is 90 deg.
The angle RQS is 20 deg.
In the triangle , the sum of the angles of the triangle is 180 deg.
\(\angle RSQ+\angle RQS+\angle SRQ=180^{\circ}\)\(90^{\circ}+20^{\circ}+\angle SRQ=180^{\circ}\)\(\angle SRQ=180^{\circ}-110^{\circ}\)\(\angle SRQ=70^{\circ^{}}\)The correct option is D.
Drawing a number divisible by 3 then drawing a 1
Answer:
Step-by-step explanation:
The probability of drawing a number that is divisible by 3 is 1/3, since there are 3 numbers (3, 6, and 9) out of the 9 possible numbers (1, 2, 3, 4, 5, 6, 7, 8, and 9) that are divisible by 3.
The probability of drawing a 1 after drawing a number that is divisible by 3 is 1/10, since there is only one 1 among the 10 possible digits (0-9) that could be drawn after the first number.
To find the probability of both events happening together (drawing a number divisible by 3 and then drawing a 1), we need to multiply the probability of the first event by the probability of the second event. Therefore, the probability of drawing a number divisible by 3 and then drawing a 1 is:
(1/3) x (1/10) = 1/30
So the probability of drawing a number divisible by 3 and then drawing a 1 is 1/30, or approximately 0.0333, or about 3.33%
Answer:
If I understand this correctly you can draw a 9 and then draw the one if I'm wrong just delete this
Step-by-step explanation:
find the sum of 2x ÷ x-1 and x÷x-1
Answer with step-by-step explanation:
First of all, ORDER OF OPERATIONS!! 2x/x is 2/1, which is 2. Then, subtract 1, which is 1.
2x ÷ x - 1 = 1
Same thing: x/x = 1. Subtract 1 from 1, which is 0.
x ÷ x - 1 = 0
1 + 0 = 1
Hope this helped!
A golfer takes an uphill shot towards the hole, and they make the shot. The path of the ball can be modeled by the equation y=-5^2– 40x– 10, where x is the horizontal distance and y is the vertical distance traveled by the ball in meters. How much higher is the hole than the spot from where the shot was taken? The hole is meters higher What was the maximum height above the hole that the ball reached while it was in the air? meters The maximum height that the ball reached above the hole was What was the horizontal distance that the ball traveled from the point that the shot was taken to the hole? Round your ai the nearest tenth of a meter. meters. The horizontal distance covered by the ball was
Answer:
Solve for n
n=log5 (− 2 /y −20x−5),y<−40x−10
Step-by-step explanation:
An angle measure is 4 more than twice it’s complement. What would it’s supplement measure?
The measure οf the supplementary angle οf x = 61.33° is fοund tο be 118.67°.
What is an angle?An angle is a figure in plane geοmetry that is created by twο rays οr lines that have a shared endpοint. The twο rays are termed the sides οf an angle, and the cοmmοn terminatiοn is called the vertex. Angles are typically expressed as degrees. A "prοtractοr" is a crucial geοmetrical instrument that aids in measuring angles in degrees. Twο sets οf numbers οn a prοtractοr are οriented in οppοsitiοn tο οne anοther. On the οutside rim, οne set gοes frοm 0 tο 180 degrees, while οn the inner rim, the οther set gοes frοm 180 tο 0 degrees.
Let the angle be x.
Twο angles are cοmplementary when their sum is 90°.
Tοw angles are supplementary when their sum is 180°
If x is οne angle, then its cοmplementary angle is (90-x)°
Given x is 4 mοre than twice its cοmplement.
We can write the equatiοn as:
x = 4 + 2*(90-x)
Sοlving,
x = 4 + 180 - 2x
3x = 184
x = 61.33°
Nοw we are asked tο find the supplementary angle οf x.
Supplementary angle = 180 - x = 180 - 61.33 = 118.67°
Therefοre the measure οf the supplementary angle οf x = 61.33° is fοund tο be 118.67°.
To learn more about angles, follow the link.
https://brainly.com/question/30111037
#SPJ1